If $a+b+c=1$ and a,b,c >0 prove $dfracb^2a+b^2+dfracc^2b+c^2+dfraca^2c+a^2 geqslant dfrac34$ [duplicate]inequality with three variables and conditionHow prove this inequality $dfracRrgedfracbc+dfraccb$Symmetric inequality for a rational function of three variablessymetric inequality for a rational function of three variablesInequality for a rational function of three variablesIf $xyz=1$, prove $frac1y(x+y)+frac1z(y+z)+frac1x(z+x) geqslant frac32$Prove: $abcgeqslant 162$How to prove $(frac 1n)^n+(frac 2n)^n+cdots+(frac nn)^ngeqslantfrac3n+12n+2$inequality with three variables and conditionInequality with condition $a^2+b^2+c^2=3$Proving that $dfracabc^3+dfracbca^3+dfraccab^3> dfrac1a+dfrac1b+dfrac1c$

Animated Series: Alien black spider robot crashes on Earth

To string or not to string

"to be prejudice towards/against someone" vs "to be prejudiced against/towards someone"

Font hinting is lost in Chrome-like browsers (for some languages )

Risk of getting Chronic Wasting Disease (CWD) in the United States?

Did Shadowfax go to Valinor?

Prove that NP is closed under karp reduction?

How did the USSR manage to innovate in an environment characterized by government censorship and high bureaucracy?

Problem of parity - Can we draw a closed path made up of 20 line segments...

How does one intimidate enemies without having the capacity for violence?

What do the dots in this tr command do: tr .............A-Z A-ZA-Z <<< "JVPQBOV" (with 13 dots)

Modeling an IPv4 Address

How does strength of boric acid solution increase in presence of salicylic acid?

Can a Warlock become Neutral Good?

Why don't electron-positron collisions release infinite energy?

A newer friend of my brother's gave him a load of baseball cards that are supposedly extremely valuable. Is this a scam?

Fencing style for blades that can attack from a distance

TGV timetables / schedules?

Why doesn't Newton's third law mean a person bounces back to where they started when they hit the ground?

Can I make popcorn with any corn?

Can an x86 CPU running in real mode be considered to be basically an 8086 CPU?

The use of multiple foreign keys on same column in SQL Server

Test if tikzmark exists on same page

How much RAM could one put in a typical 80386 setup?



If $a+b+c=1$ and a,b,c >0 prove $dfracb^2a+b^2+dfracc^2b+c^2+dfraca^2c+a^2 geqslant dfrac34$ [duplicate]


inequality with three variables and conditionHow prove this inequality $dfracRrgedfracbc+dfraccb$Symmetric inequality for a rational function of three variablessymetric inequality for a rational function of three variablesInequality for a rational function of three variablesIf $xyz=1$, prove $frac1y(x+y)+frac1z(y+z)+frac1x(z+x) geqslant frac32$Prove: $abcgeqslant 162$How to prove $(frac 1n)^n+(frac 2n)^n+cdots+(frac nn)^ngeqslantfrac3n+12n+2$inequality with three variables and conditionInequality with condition $a^2+b^2+c^2=3$Proving that $dfracabc^3+dfracbca^3+dfraccab^3> dfrac1a+dfrac1b+dfrac1c$













-1












$begingroup$



This question already has an answer here:



  • inequality with three variables and condition

    1 answer



If $a+b+c=1$ and a,b,c>0 prove $dfracb^2a+b^2+dfracc^2b+c^2+dfraca^2c+a^2 geqslant dfrac34$. I tried with CS Engel form,homogenization but ina anyway i can't prove inequality. Can someone helpp?










share|cite|improve this question











$endgroup$



marked as duplicate by Martin R, Macavity inequality
Users with the  inequality badge can single-handedly close inequality questions as duplicates and reopen them as needed.

StackExchange.ready(function()
if (StackExchange.options.isMobile) return;

$('.dupe-hammer-message-hover:not(.hover-bound)').each(function()
var $hover = $(this).addClass('hover-bound'),
$msg = $hover.siblings('.dupe-hammer-message');

$hover.hover(
function()
$hover.showInfoMessage('',
messageElement: $msg.clone().show(),
transient: false,
position: my: 'bottom left', at: 'top center', offsetTop: -7 ,
dismissable: false,
relativeToBody: true
);
,
function()
StackExchange.helpers.removeMessages();

);
);
);
Mar 22 at 8:52


This question has been asked before and already has an answer. If those answers do not fully address your question, please ask a new question.













  • 1




    $begingroup$
    You asked the identical question three years ago (and got an answer) ...
    $endgroup$
    – Martin R
    Mar 22 at 8:39















-1












$begingroup$



This question already has an answer here:



  • inequality with three variables and condition

    1 answer



If $a+b+c=1$ and a,b,c>0 prove $dfracb^2a+b^2+dfracc^2b+c^2+dfraca^2c+a^2 geqslant dfrac34$. I tried with CS Engel form,homogenization but ina anyway i can't prove inequality. Can someone helpp?










share|cite|improve this question











$endgroup$



marked as duplicate by Martin R, Macavity inequality
Users with the  inequality badge can single-handedly close inequality questions as duplicates and reopen them as needed.

StackExchange.ready(function()
if (StackExchange.options.isMobile) return;

$('.dupe-hammer-message-hover:not(.hover-bound)').each(function()
var $hover = $(this).addClass('hover-bound'),
$msg = $hover.siblings('.dupe-hammer-message');

$hover.hover(
function()
$hover.showInfoMessage('',
messageElement: $msg.clone().show(),
transient: false,
position: my: 'bottom left', at: 'top center', offsetTop: -7 ,
dismissable: false,
relativeToBody: true
);
,
function()
StackExchange.helpers.removeMessages();

);
);
);
Mar 22 at 8:52


This question has been asked before and already has an answer. If those answers do not fully address your question, please ask a new question.













  • 1




    $begingroup$
    You asked the identical question three years ago (and got an answer) ...
    $endgroup$
    – Martin R
    Mar 22 at 8:39













-1












-1








-1


0



$begingroup$



This question already has an answer here:



  • inequality with three variables and condition

    1 answer



If $a+b+c=1$ and a,b,c>0 prove $dfracb^2a+b^2+dfracc^2b+c^2+dfraca^2c+a^2 geqslant dfrac34$. I tried with CS Engel form,homogenization but ina anyway i can't prove inequality. Can someone helpp?










share|cite|improve this question











$endgroup$





This question already has an answer here:



  • inequality with three variables and condition

    1 answer



If $a+b+c=1$ and a,b,c>0 prove $dfracb^2a+b^2+dfracc^2b+c^2+dfraca^2c+a^2 geqslant dfrac34$. I tried with CS Engel form,homogenization but ina anyway i can't prove inequality. Can someone helpp?





This question already has an answer here:



  • inequality with three variables and condition

    1 answer







inequality cauchy-schwarz-inequality rearrangement-inequality






share|cite|improve this question















share|cite|improve this question













share|cite|improve this question




share|cite|improve this question








edited Mar 21 at 23:34







chaos

















asked Mar 21 at 21:58









chaoschaos

23918




23918




marked as duplicate by Martin R, Macavity inequality
Users with the  inequality badge can single-handedly close inequality questions as duplicates and reopen them as needed.

StackExchange.ready(function()
if (StackExchange.options.isMobile) return;

$('.dupe-hammer-message-hover:not(.hover-bound)').each(function()
var $hover = $(this).addClass('hover-bound'),
$msg = $hover.siblings('.dupe-hammer-message');

$hover.hover(
function()
$hover.showInfoMessage('',
messageElement: $msg.clone().show(),
transient: false,
position: my: 'bottom left', at: 'top center', offsetTop: -7 ,
dismissable: false,
relativeToBody: true
);
,
function()
StackExchange.helpers.removeMessages();

);
);
);
Mar 22 at 8:52


This question has been asked before and already has an answer. If those answers do not fully address your question, please ask a new question.









marked as duplicate by Martin R, Macavity inequality
Users with the  inequality badge can single-handedly close inequality questions as duplicates and reopen them as needed.

StackExchange.ready(function()
if (StackExchange.options.isMobile) return;

$('.dupe-hammer-message-hover:not(.hover-bound)').each(function()
var $hover = $(this).addClass('hover-bound'),
$msg = $hover.siblings('.dupe-hammer-message');

$hover.hover(
function()
$hover.showInfoMessage('',
messageElement: $msg.clone().show(),
transient: false,
position: my: 'bottom left', at: 'top center', offsetTop: -7 ,
dismissable: false,
relativeToBody: true
);
,
function()
StackExchange.helpers.removeMessages();

);
);
);
Mar 22 at 8:52


This question has been asked before and already has an answer. If those answers do not fully address your question, please ask a new question.









  • 1




    $begingroup$
    You asked the identical question three years ago (and got an answer) ...
    $endgroup$
    – Martin R
    Mar 22 at 8:39












  • 1




    $begingroup$
    You asked the identical question three years ago (and got an answer) ...
    $endgroup$
    – Martin R
    Mar 22 at 8:39







1




1




$begingroup$
You asked the identical question three years ago (and got an answer) ...
$endgroup$
– Martin R
Mar 22 at 8:39




$begingroup$
You asked the identical question three years ago (and got an answer) ...
$endgroup$
– Martin R
Mar 22 at 8:39










1 Answer
1






active

oldest

votes


















2












$begingroup$

It's wrong for real numbers.



For positive variables by C-S
$$sum_cycfraca^2a^2+c=sum_cycfraca^2(a+b)^2(c(a+b+c)+a^2)(a+b)^2geqfracleft(sumlimits_cyc(a^2+ab)right)^2sumlimits_cyc(a^2+c^2+ac+bc)(a+b)^2.$$
Thus, it's enough to prove that
$$4left(sumlimits_cyc(a^2+ab)right)^2geq3sumlimits_cyc(a^2+c^2+ac+bc)(a+b)^2$$ or
$$sum_cyc(a^4-a^3b+5a^3c+3a^2b^2-8a^2bc)geq0,$$ which is true because
$$sum_cyc(a^4-a^3b)geq0$$ by Rearrangement;
$$sum_cyca^3cgeqsum_cyca^2bc$$ it's
$$sum_cycfraca^2bgeqsum_cyca,$$ which is true by Rearrangement again and
$$sum_cyc(a^2b^2-a^2bc)=frac12sum_cycc^2(a-b)^2geq0.$$






share|cite|improve this answer









$endgroup$












  • $begingroup$
    +1 for the good efforts, even though duplicate.
    $endgroup$
    – Macavity
    Mar 22 at 8:53










  • $begingroup$
    @Macavity: Michael already answered the original question: math.stackexchange.com/a/1602340/42969 :)
    $endgroup$
    – Martin R
    Mar 22 at 8:54











  • $begingroup$
    @Macavity Thank you! I usually don't remember what I proved.
    $endgroup$
    – Michael Rozenberg
    Mar 22 at 9:36










  • $begingroup$
    @MichaelRozenberg: It is quite easy with Approach0 – it takes less than a minute to find that this question has been asked and answered before.
    $endgroup$
    – Martin R
    Mar 22 at 10:06


















1 Answer
1






active

oldest

votes








1 Answer
1






active

oldest

votes









active

oldest

votes






active

oldest

votes









2












$begingroup$

It's wrong for real numbers.



For positive variables by C-S
$$sum_cycfraca^2a^2+c=sum_cycfraca^2(a+b)^2(c(a+b+c)+a^2)(a+b)^2geqfracleft(sumlimits_cyc(a^2+ab)right)^2sumlimits_cyc(a^2+c^2+ac+bc)(a+b)^2.$$
Thus, it's enough to prove that
$$4left(sumlimits_cyc(a^2+ab)right)^2geq3sumlimits_cyc(a^2+c^2+ac+bc)(a+b)^2$$ or
$$sum_cyc(a^4-a^3b+5a^3c+3a^2b^2-8a^2bc)geq0,$$ which is true because
$$sum_cyc(a^4-a^3b)geq0$$ by Rearrangement;
$$sum_cyca^3cgeqsum_cyca^2bc$$ it's
$$sum_cycfraca^2bgeqsum_cyca,$$ which is true by Rearrangement again and
$$sum_cyc(a^2b^2-a^2bc)=frac12sum_cycc^2(a-b)^2geq0.$$






share|cite|improve this answer









$endgroup$












  • $begingroup$
    +1 for the good efforts, even though duplicate.
    $endgroup$
    – Macavity
    Mar 22 at 8:53










  • $begingroup$
    @Macavity: Michael already answered the original question: math.stackexchange.com/a/1602340/42969 :)
    $endgroup$
    – Martin R
    Mar 22 at 8:54











  • $begingroup$
    @Macavity Thank you! I usually don't remember what I proved.
    $endgroup$
    – Michael Rozenberg
    Mar 22 at 9:36










  • $begingroup$
    @MichaelRozenberg: It is quite easy with Approach0 – it takes less than a minute to find that this question has been asked and answered before.
    $endgroup$
    – Martin R
    Mar 22 at 10:06
















2












$begingroup$

It's wrong for real numbers.



For positive variables by C-S
$$sum_cycfraca^2a^2+c=sum_cycfraca^2(a+b)^2(c(a+b+c)+a^2)(a+b)^2geqfracleft(sumlimits_cyc(a^2+ab)right)^2sumlimits_cyc(a^2+c^2+ac+bc)(a+b)^2.$$
Thus, it's enough to prove that
$$4left(sumlimits_cyc(a^2+ab)right)^2geq3sumlimits_cyc(a^2+c^2+ac+bc)(a+b)^2$$ or
$$sum_cyc(a^4-a^3b+5a^3c+3a^2b^2-8a^2bc)geq0,$$ which is true because
$$sum_cyc(a^4-a^3b)geq0$$ by Rearrangement;
$$sum_cyca^3cgeqsum_cyca^2bc$$ it's
$$sum_cycfraca^2bgeqsum_cyca,$$ which is true by Rearrangement again and
$$sum_cyc(a^2b^2-a^2bc)=frac12sum_cycc^2(a-b)^2geq0.$$






share|cite|improve this answer









$endgroup$












  • $begingroup$
    +1 for the good efforts, even though duplicate.
    $endgroup$
    – Macavity
    Mar 22 at 8:53










  • $begingroup$
    @Macavity: Michael already answered the original question: math.stackexchange.com/a/1602340/42969 :)
    $endgroup$
    – Martin R
    Mar 22 at 8:54











  • $begingroup$
    @Macavity Thank you! I usually don't remember what I proved.
    $endgroup$
    – Michael Rozenberg
    Mar 22 at 9:36










  • $begingroup$
    @MichaelRozenberg: It is quite easy with Approach0 – it takes less than a minute to find that this question has been asked and answered before.
    $endgroup$
    – Martin R
    Mar 22 at 10:06














2












2








2





$begingroup$

It's wrong for real numbers.



For positive variables by C-S
$$sum_cycfraca^2a^2+c=sum_cycfraca^2(a+b)^2(c(a+b+c)+a^2)(a+b)^2geqfracleft(sumlimits_cyc(a^2+ab)right)^2sumlimits_cyc(a^2+c^2+ac+bc)(a+b)^2.$$
Thus, it's enough to prove that
$$4left(sumlimits_cyc(a^2+ab)right)^2geq3sumlimits_cyc(a^2+c^2+ac+bc)(a+b)^2$$ or
$$sum_cyc(a^4-a^3b+5a^3c+3a^2b^2-8a^2bc)geq0,$$ which is true because
$$sum_cyc(a^4-a^3b)geq0$$ by Rearrangement;
$$sum_cyca^3cgeqsum_cyca^2bc$$ it's
$$sum_cycfraca^2bgeqsum_cyca,$$ which is true by Rearrangement again and
$$sum_cyc(a^2b^2-a^2bc)=frac12sum_cycc^2(a-b)^2geq0.$$






share|cite|improve this answer









$endgroup$



It's wrong for real numbers.



For positive variables by C-S
$$sum_cycfraca^2a^2+c=sum_cycfraca^2(a+b)^2(c(a+b+c)+a^2)(a+b)^2geqfracleft(sumlimits_cyc(a^2+ab)right)^2sumlimits_cyc(a^2+c^2+ac+bc)(a+b)^2.$$
Thus, it's enough to prove that
$$4left(sumlimits_cyc(a^2+ab)right)^2geq3sumlimits_cyc(a^2+c^2+ac+bc)(a+b)^2$$ or
$$sum_cyc(a^4-a^3b+5a^3c+3a^2b^2-8a^2bc)geq0,$$ which is true because
$$sum_cyc(a^4-a^3b)geq0$$ by Rearrangement;
$$sum_cyca^3cgeqsum_cyca^2bc$$ it's
$$sum_cycfraca^2bgeqsum_cyca,$$ which is true by Rearrangement again and
$$sum_cyc(a^2b^2-a^2bc)=frac12sum_cycc^2(a-b)^2geq0.$$







share|cite|improve this answer












share|cite|improve this answer



share|cite|improve this answer










answered Mar 21 at 22:37









Michael RozenbergMichael Rozenberg

110k1896201




110k1896201











  • $begingroup$
    +1 for the good efforts, even though duplicate.
    $endgroup$
    – Macavity
    Mar 22 at 8:53










  • $begingroup$
    @Macavity: Michael already answered the original question: math.stackexchange.com/a/1602340/42969 :)
    $endgroup$
    – Martin R
    Mar 22 at 8:54











  • $begingroup$
    @Macavity Thank you! I usually don't remember what I proved.
    $endgroup$
    – Michael Rozenberg
    Mar 22 at 9:36










  • $begingroup$
    @MichaelRozenberg: It is quite easy with Approach0 – it takes less than a minute to find that this question has been asked and answered before.
    $endgroup$
    – Martin R
    Mar 22 at 10:06

















  • $begingroup$
    +1 for the good efforts, even though duplicate.
    $endgroup$
    – Macavity
    Mar 22 at 8:53










  • $begingroup$
    @Macavity: Michael already answered the original question: math.stackexchange.com/a/1602340/42969 :)
    $endgroup$
    – Martin R
    Mar 22 at 8:54











  • $begingroup$
    @Macavity Thank you! I usually don't remember what I proved.
    $endgroup$
    – Michael Rozenberg
    Mar 22 at 9:36










  • $begingroup$
    @MichaelRozenberg: It is quite easy with Approach0 – it takes less than a minute to find that this question has been asked and answered before.
    $endgroup$
    – Martin R
    Mar 22 at 10:06
















$begingroup$
+1 for the good efforts, even though duplicate.
$endgroup$
– Macavity
Mar 22 at 8:53




$begingroup$
+1 for the good efforts, even though duplicate.
$endgroup$
– Macavity
Mar 22 at 8:53












$begingroup$
@Macavity: Michael already answered the original question: math.stackexchange.com/a/1602340/42969 :)
$endgroup$
– Martin R
Mar 22 at 8:54





$begingroup$
@Macavity: Michael already answered the original question: math.stackexchange.com/a/1602340/42969 :)
$endgroup$
– Martin R
Mar 22 at 8:54













$begingroup$
@Macavity Thank you! I usually don't remember what I proved.
$endgroup$
– Michael Rozenberg
Mar 22 at 9:36




$begingroup$
@Macavity Thank you! I usually don't remember what I proved.
$endgroup$
– Michael Rozenberg
Mar 22 at 9:36












$begingroup$
@MichaelRozenberg: It is quite easy with Approach0 – it takes less than a minute to find that this question has been asked and answered before.
$endgroup$
– Martin R
Mar 22 at 10:06





$begingroup$
@MichaelRozenberg: It is quite easy with Approach0 – it takes less than a minute to find that this question has been asked and answered before.
$endgroup$
– Martin R
Mar 22 at 10:06




Popular posts from this blog

Lowndes Grove History Architecture References Navigation menu32°48′6″N 79°57′58″W / 32.80167°N 79.96611°W / 32.80167; -79.9661132°48′6″N 79°57′58″W / 32.80167°N 79.96611°W / 32.80167; -79.9661178002500"National Register Information System"Historic houses of South Carolina"Lowndes Grove""+32° 48' 6.00", −79° 57' 58.00""Lowndes Grove, Charleston County (260 St. Margaret St., Charleston)""Lowndes Grove"The Charleston ExpositionIt Happened in South Carolina"Lowndes Grove (House), Saint Margaret Street & Sixth Avenue, Charleston, Charleston County, SC(Photographs)"Plantations of the Carolina Low Countrye

random experiment with two different functions on unit interval Announcing the arrival of Valued Associate #679: Cesar Manara Planned maintenance scheduled April 23, 2019 at 00:00UTC (8:00pm US/Eastern)Random variable and probability space notionsRandom Walk with EdgesFinding functions where the increase over a random interval is Poisson distributedNumber of days until dayCan an observed event in fact be of zero probability?Unit random processmodels of coins and uniform distributionHow to get the number of successes given $n$ trials , probability $P$ and a random variable $X$Absorbing Markov chain in a computer. Is “almost every” turned into always convergence in computer executions?Stopped random walk is not uniformly integrable

How should I support this large drywall patch? Planned maintenance scheduled April 23, 2019 at 00:00UTC (8:00pm US/Eastern) Announcing the arrival of Valued Associate #679: Cesar Manara Unicorn Meta Zoo #1: Why another podcast?How do I cover large gaps in drywall?How do I keep drywall around a patch from crumbling?Can I glue a second layer of drywall?How to patch long strip on drywall?Large drywall patch: how to avoid bulging seams?Drywall Mesh Patch vs. Bulge? To remove or not to remove?How to fix this drywall job?Prep drywall before backsplashWhat's the best way to fix this horrible drywall patch job?Drywall patching using 3M Patch Plus Primer